Difference between revisions of "2017 AMC 12A Problems/Problem 10"

(Solution)
(Alternate Solution: Geometric Probability)
Line 9: Line 9:
  
 
==Alternate Solution: Geometric Probability==
 
==Alternate Solution: Geometric Probability==
Let <math>x</math> be the number chosen randomly by Chloé. Because it is given that the number Chloé choose is interval <math> [ 0, 2017 ] </math>, <math> 0 \leq x \leq 2017</math>.  Next, let <math>y</math>  be the number chosen randomly by Laurent. Because it is given that the number Chloé choose is interval <math> [ 0, 2017 ] </math>, <math> 0 \leq y \leq 4034</math>. Since we are looking for when Laurent's number is Chloé's we write the equation <math>y > x</math>. When these three inequalities are graphed the area captured by <math> 0 \leq x \leq 2017</math> and <math> 0 \leq y \leq 4034</math> represents the all the possibilities and the area captured by <math> 0 \leq x \leq 2017</math>, <math> 0 \leq y \leq 4034</math>, and <math>y > x</math> represents the possibilities of Laurent winning.  The simplified quotient of these two areas is the probability Laurent's number is larger than Chloé's, which is <math>\boxed \dfrac{3}{4} = C</math>.
+
Let <math>x</math> be the number chosen randomly by Chloé. Because it is given that the number Chloé choose is interval <math> [ 0, 2017 ] </math>, <math> 0 \leq x \leq 2017</math>.  Next, let <math>y</math>  be the number chosen randomly by Laurent. Because it is given that the number Chloé choose is interval <math> [ 0, 2017 ] </math>, <math> 0 \leq y \leq 4034</math>. Since we are looking for when Laurent's number is Chloé's we write the equation <math>y > x</math>. When these three inequalities are graphed the area captured by <math> 0 \leq x \leq 2017</math> and <math> 0 \leq y \leq 4034</math> represents the all the possibilities and the area captured by <math> 0 \leq x \leq 2017</math>, <math> 0 \leq y \leq 4034</math>, and <math>y > x</math> represents the possibilities of Laurent winning.  The simplified quotient of these two areas is the probability Laurent's number is larger than Chloé's, which is <math> \frac{3}{4} = C</math>.
  
 
==See Also==
 
==See Also==

Revision as of 16:19, 9 February 2017

Problem

Chloé chooses a real number uniformly at random from the interval $[ 0,2017 ]$. Independently, Laurent chooses a real number uniformly at random from the interval $[ 0 , 4034 ]$. What is the probability that Laurent's number is greater than Chloe's number?

$\textbf{(A)}\ \dfrac{1}{2} \qquad\textbf{(B)}\ \dfrac{2}{3} \qquad\textbf{(C)}\ \dfrac{3}{4} \qquad\textbf{(D)}\ \dfrac{5}{6} \qquad\textbf{(E)}\ \dfrac{7}{8}$

Solution

Suppose Laurent's number is in the interval $[ 0, 2017 ]$. Then, by symmetry, the probability of Laurent's number being greater is $\dfrac{1}{2}$. Next, suppose Laurent's number is in the interval $[ 2017, 4034 ]$. Then Laurent's number will be greater with probability $1$. Since each case is equally likely, the probability of Laurent's number being greater is $\dfrac{1 + \frac{1}{2}}{2} = \dfrac{3}{4}$, so the answer is C.

Alternate Solution: Geometric Probability

Let $x$ be the number chosen randomly by Chloé. Because it is given that the number Chloé choose is interval $[ 0, 2017 ]$, $0 \leq x \leq 2017$. Next, let $y$ be the number chosen randomly by Laurent. Because it is given that the number Chloé choose is interval $[ 0, 2017 ]$, $0 \leq y \leq 4034$. Since we are looking for when Laurent's number is Chloé's we write the equation $y > x$. When these three inequalities are graphed the area captured by $0 \leq x \leq 2017$ and $0 \leq y \leq 4034$ represents the all the possibilities and the area captured by $0 \leq x \leq 2017$, $0 \leq y \leq 4034$, and $y > x$ represents the possibilities of Laurent winning. The simplified quotient of these two areas is the probability Laurent's number is larger than Chloé's, which is $\frac{3}{4} = C$.

See Also

2017 AMC 10A (ProblemsAnswer KeyResources)
Preceded by
Problem 14
Followed by
Problem 16
1 2 3 4 5 6 7 8 9 10 11 12 13 14 15 16 17 18 19 20 21 22 23 24 25
All AMC 10 Problems and Solutions
2017 AMC 12A (ProblemsAnswer KeyResources)
Preceded by
Problem 9
Followed by
Problem 11
1 2 3 4 5 6 7 8 9 10 11 12 13 14 15 16 17 18 19 20 21 22 23 24 25
All AMC 12 Problems and Solutions

The problems on this page are copyrighted by the Mathematical Association of America's American Mathematics Competitions. AMC logo.png